block 4 exam 4 -- Mega Practice Q's

अब Quizwiz के साथ अपने होमवर्क और परीक्षाओं को एस करें!

The answer is E: Smooth muscle hyperplasia and basement mem-brane thickening. When severe acute asthma is unresponsive to therapy, it is referred to as status asthmaticus. Histological examination of lung from a patient who died in status asth-maticus often shows a bronchus containing a luminal mucous plug, submucosal gland hyperplasia, smooth muscle hyperpla-sia, basement membrane thickening, and increased numbers of eosinophils. All of the other choices concern alveolar damage, whereas the photograph demonstrates a section of bronchus. Diagnosis: Asthma

A 10-year-old boy dies following a severe episode of status asthmaticus. Histologic examination of the lung at autopsy is shown in the image. Which of the following best describes the pathologic features evident in this autopsy specimen? (A) Destruction of the walls of airspaces without fi brosis (B) Hyaline membranes and interstitial edema (C) Interstitial fi brosis of the lung parenchyma (D) Intra-alveolar hemorrhage and exudates containing neutrophils (E) Smooth muscle hyperplasia and basement membrane thickening

The answer is B: Goodpasture syndrome. Goodpasture syndrome is an autoimmune disease in which autoantibodies bind to the noncollagenous domain of type IV collagen. This connective tissue protein is a major structural component of both pulmonary and glomerular basement membranes. Local complement activation results in the recruitment of neutro-phils, tissue injury, pulmonary hemorrhage, and glomerulone-phritis. Anti-type IV collagen antibodies are not encountered in the other choices. Diagnosis: Goodpasture syndrom

A 22-year-old man presents with a 6-month history of increas-ing shortness of breath and persistent cough with rusty spu-tum. A chest X-ray shows diffuse bilateral alveolar infi ltrates. Urine dipstick analysis reveals 2+ hematuria. A transbronchial lung biopsy is shown in the image. Linear deposits of IgG and complement C3 are detected in the alveolar basement mem-brane by immunofl uorescence. Which of the following is the most likely diagnosis? (A) Churg-Strauss syndrome (B) Goodpasture syndrome (C) Hypersensitivity pneumonitis (D) Loeffl er syndrome (E) Wegener granulomatosis

The answer is A: Alveolar proteinosis. Alveolar proteinosis (also termed lipoproteinosis) is a rare condition in which the alveoli are fi lled with a granular, proteinaceous, eosinophilic material, which is PAS-positive, diastase resistant, and rich in lipids. The disease was initially described as idiopathic, but recent studies have associated alveolar proteinosis with com-promised immunity, leukemia and lymphoma, respiratory infections, and exposure to environmental inorganic dusts. Repeated bronchoalveolar lavage is used to remove the alveo-lar material, and repeated lavage may halt progression of the disease. None of the other choices exhibit an acellular eosino-philic material that fi lls the alveoli. Diagnosis: Alveolar proteinosis

A 22-year-old man who is being treated for leukemia complains of shortness of breath on exertion, pleuritic chest pain, and a low-grade fever. Physical examination reveals crackles in both lung bases and clubbing of the fi ngers. Bronchoalveolar lavage demonstrates PAS-positive mate-rial and elevated levels of surfactant proteins. An open-lung biopsy is shown in the image. Which of the following is the most likely diagnosis? (A) Alveolar proteinosis (B) Eosinophilic pneumonia (C) Goodpasture syndrome (D) Hyaline membrane disease (E) Radiation pneumonitis

The answer is B: Asbestosis. Asbestosis refers to the diffuse interstitial fi brosis that results from the inhalation of asbestos fi bers. The disease occurs as a result of the processing and handling of asbestos, rather than mining, which is a surface operation. Asbestosis is characterized by bilateral, diffuse interstitial fi brosis and asbestos bodies in the lung. These fer-ruginous bodies are golden brown and beaded, with a cen-tral colorless core fi ber. Asbestos bodies are encrusted with protein and iron. In this patient, the dome of the diaphragm is covered by a smooth, pearly white, nodular fi brotic lesion (pleural plaque), a common feature of asbestos exposure. A clear-cut relationship between occupational asbestos expo-sure and malignant mesothelioma is established. None of the other choices display pleural plaques or ferruginous bodies. Diagnosis: Asbestosis

A 75-year-old man who had worked in a shipyard dies of a chronic lung disease. Autopsy reveals extensive pulmonary fi brosis, and iron stains of lung tissue show numerous fer-ruginous bodies. The dome of the diaphragm is shown in the image. What is the appropriate diagnosis? (A) Anthracosis (B) Asbestosis (C) Berylliosis (D) Sarcoidosis (E) Silicosis

The answer is A. In a typical well-differentiated bronchioloalveolar carcinoma, tumor cells line the walls of terminal air spaces, as shown in the illustration. When the tumor is localized to a single nodule, it is potentially curable by surgical resection

A single nodule was resected from the peripheral portion of the right lower lobe of the lung of a 45-year-old woman. The micro-scopic findings were similar to those shown in the illustration. The diagnosis is (A)bronchioloalveolar carcinoma. (B)carcinoid. (C)mesothelioma. (D)small cell carcinoma. (E)squamous cell carcinoma.

The answer is E: Wegener granulomatosis (WG). WG is a disease of unknown cause that is characterized by aseptic, necrotizing, granulomatous infl ammation and vasculitis. This disease affects the upper and lower respiratory tract and kidneys. Pulmonary features of WG include necrotizing granulomatous infl amma-tion, parenchymal necrosis, and vasculitis. In most cases, mul-tiple nodules averaging 2 to 3 cm in diameter are seen in the lungs. WG most commonly affects the head and neck, followed by the lung, kidney, and eye. Respiratory manifestations include sinusitis, cough, hemoptysis, and pleuritis. Sinus involvement is not common in the incorrect choices. Churg-Strauss syndrome (choice B) shares some features with WG, but is characterized by asthma, peripheral eosinophilia and P-ANCA. Diagnosis: Wegener granulomatosis

A 23-year-old man complains of nasal obstruction, sero-sanguinous discharge, cough, and bloody sputum. A chest X-ray shows cavitated lesions and multiple nodules over both lung fi elds. A CT scan discloses obliteration of several maxillary sinuses. Urinalysis reveals hematuria and RBC casts. Laboratory studies demonstrate anemia and elevated serum levels of C-ANCA. An open-lung biopsy is shown in the image. Which of the following is the most likely diagnosis? (A) Adenocarcinoma of lung (B) Churg-Strauss syndrome (C) Necrotizing sarcoid granulomatosis (D) Tuberculosis (E) Wegener granulomatosis

the answer is c. The illustration shows noncaseating granulomas and giant cells, which, in the clinical setting described, are diagnostic of sarcoidosis. A frequent abnormal labo-ratory finding is polyclonal hypergammaglobulinemia along with hypercalcemia. Anergy to tuberculin is often demonstrable. The disorder is much more common in persons of African lineage. Patients most often present with lung findings and hilar lymphadenopa-thy, but any organ system can be involved

A 23-year-old man presents with radiographic evidence of bilateral hilar lymphadenopathy and interstitial lung disease. A lung biopsy gives findings similar to those shown in the figure. A major characteristic of this disorder is a)a positive test for tuberculin. (B)highest incidence in persons of Asian lineage (c)hypercalcemia. (D)hypogammaglobulinemia. (e)involvement restricted to the lung.

The answer is d. Sarcoidosis is a systemic disease characterized by noncaseating granulomas in multipleorgans. The diagnosis of sarcoidosis depends on finding these noncaseating granulomas in commonly affected sites. In 90% of cases, bilateral hilar lymphadenopathy ("potatonodes") or lung involvement is present and can be revealed by chest x-ray or transbronchial biopsy. The eye and skin are the next most commonly affected organs, so thatboth conjunctival and skin biopsies are clinical possibilities. Noncaseating granulomas may be found in multiple infectious diseases, such as fungal infections, but sarcoidosis isnot caused by any known organism. Therefore, before the diagnosis of sarcoidosis can be made, cultures must be taken from affected tissues, and there must be no growth ofany organism that may produce granulomas. In patients with sarcoidosis, blood levels of angiotensin-converting enzyme are increased, and this may also be used as a clinicaltest. In the past, the Kveim skin test was used to assist in the diagnosis of sarcoidosis, but because it involves injecting into patients extracts of material from humans, it is nolong

A 24-year-old African American woman presents with nonspecific symptoms including fever and malaise. A chest x-ray reveals enlarged hilar lymph nodes ("potatonodes"), while her serum calcium level is found to be elevated. Which of the following histologic abnormalities is most likely to be seen in biopsy specimens from theseenlarged hilar lymph nodes? a. Caseating granulomas b. Dense, granular, PAS-positive, eosinophilic material c. Markedly enlarged epithelial cells with intranuclear inclusions d. Noncaseating granulomas e. Numerous neutrophils with fibrin deposition

The answer is A. The combination of emphysema and hepatic cirrhosis in a young per-son with a family history of similarly affected family members is strongly suggestive of homozygous α1-antitrypsin deficiency. In these instances the emphysema is usually panacinar in type

A 24-year-old nonsmoker who does not consume alcohol is found to have both pulmonary emphysema and cirrhosis of the liver. A sister and several close relatives also have had similar findings. This history suggests that this patient's illness may be caused by deficiency of (A)α1-antitrypsin. (B)galactokinase. (C)glucose-6-phosphatase. (D)glucocerebrosidase. (E)phenylalanine hydroxylase

the answer is e. Sarcoidosis most often presents as a restrictive pulmonary disease that is characterized morphologically by noncaseating granulomas and can involve any organ system. Diagnostic features of note include highest incidence in persons of African lin-eage, somewhat higher incidence in women, bilateral interstitial pulmonary involvement, prominent hilar lymphadenopathy, polyclonal hypergammaglobulinemia, and hypercal-cemia. Increased serum angiotensin-converting enzyme activity is a nonspecific indicator of granulomatous inflammation.

A 25-year-old African-American woman presents with fatigue, dyspnea, nonpro-ductive cough, and chest pain. She does not smoke. A chest radiograph reveals prominent bilateral hilar lymphadenopa-thy ("potato nodules") and diffuse reticular densities in the interstitium of the lung. Laboratory studies reveal polyclonal hyper-gammaglobulinemia, hypercalcemia, and increased serum angiotensin-converting enzyme. Which of the following is the most likely diagnosis? (a)Acute respiratory distress syndrome (B)Adenocarcinoma of the lung (c)Eosinophilic granuloma (D)Idiopathic pulmonary fibrosis (e)Sarcoidosis

The answer is B: Sarcoidosis. Sarcoidosis is a granulomatous disease of unknown etiology. In sarcoidosis, the lung is the most frequently involved organ, but the lymph nodes, skin, and eyes are also common targets. Angiotensin-converting enzyme (ACE) is produced by epithelioid macrophages and is elevated in the blood. Spontaneous regression of lesions is common, but in some cases, the disease causes pulmonary fi brosis and respiratory failure. Symptoms of pulmonary dis-ease include dyspnea, cough, and wheezing. None of the other choices are associated with increased serum levels of ACE. Diagnosis: Sarcoidosis

A 25-year-old black woman presents with a 3-month history of cough and shortness of breath on exertion. A chest X-ray reveals enlargement of hilar and mediastinal lymph nodes. Laboratory studies show elevated serum levels of angiotensin-converting enzyme and an increase in 24-hour urine calcium excretion. An open-lung biopsy is shown in the image. Stains for microorganisms in the tissue are negative. Which of the following is the most likely diagnosis? (A) Goodpasture syndrome (B) Sarcoidosis (C) Silicosis (D) Tuberculosis (E) Wegener granulomatosis

The answer is d. Within the lungs the plexiform lesion, which is a vascular lesion that consists ofintraluminal angiomatous tufts forming webs, is thought to be diagnostic of pulmonary hypertension. Elevation of the mean pulmonary arterial pressure is the result ofvascular changes and endothelial dysfunction. The vascular changes associated with pulmonary hypertension are referred to as pulmonary vascular sclerosis. These vascularchanges vary with the size of the vessel. The main arteries have atheromas that are similar to systemic atherosclerosis, but are not as severe. Medium-sized arteries showintimal thickening and neomuscularization. Smaller arteries and arterioles show intimal thickening, medial hypertrophy, and reduplication of the internal and external elasticmembranes.The changes of pulmonary vascular sclerosis may be primary or secondary. Primary pulmonary vascular sclerosis almost always occurs in young women, who developfatigue, syncope (with exercise), dyspnea on exertion (DOE), and chest pain. Primary pulmonary hypertension is caused by mutations involving the bone morphogeneticprotein receptor type II (BMPR2) sign

A 25-year-old woman presents with a 6-month history of increasing fatigue and dyspnea. Physical examination finds that she is in moderate respiratory distress andcyanosis is present. An echocardiogram of her heart finds the thickness of the right ventricle to be increased, but the thickness of the left ventricle is within normal limits.Elevation of the mean pulmonary arterial pressure is found and histologic sections from a lung biopsy reveal plexiform lesions within the pulmonary arterioles. Hyalinemembranes are not found nor are areas of lung collapse present. Extensive workup finds an abnormality involving the BMPR2 signaling pathway. What is the correctdiagnosis? a. Churg-Strauss syndrome b. Hypersensitivity pneumonitis c. Idiopathic pulmonary hemosiderosis d. Primary pulmonary hypertension e. Wegener granulomatosis

The answer is A: Eosinophilic pneumonia. Eosinophilic pneu-monia is principally an allergic disorder. It refers to the accu-mulation of eosinophils in alveolar spaces and is classifi ed as either idiopathic or secondary to an underlying illness. In acute eosinophilic pneumonia, the alveolar spaces are fi lled with an infl ammatory exudate composed of eosinophils and macrophages. The alveolar septa are thickened by the pres-ence of numerous eosinophils and hyaline membranes are present. Patients respond dramatically to corticosteroids, and, in contrast to chronic eosinophilic pneumonia, acute eosino-philic pneumonia does not recur. Excess eosinophils are not encountered in the other choices. Diagnosis: Eosinophilic pneumonia

A 28-year-old man presents with 6 days of fever and shortness of breath. His temperature is 38.7°C (103°F), respirations are 30 per minute, and blood pressure is 120/80 mm Hg. A chest X-ray reveals diffuse interstitial and alveolar infi ltrates. Spu-tum cultures are negative, and the patient does not respond to standard antibiotic therapy. A transbronchial lung biopsy is shown in the image. Which of the following is the appropriate diagnosis? (A) Eosinophilic pneumonia (B) Lipid pneumonia (C) Pneumococcal pneumonia (D) Pneumocystis pneumonia (E) Usual interstitial pneumonia

The answer is E. The clinical picture strongly suggests a diagnosis of sarcoidosis. The granulomas of sarcoidosis are characteristically noncaseating. Sarcoidosis is a multisys-tem disorder. Common findings in this highly variable disorder include anergy to tuber-culin, hypercalcemia, and broad-based polyclonal hypergammaglobulinemia.

A 30-year-old African-American womanpresents with bilateral hilar lymphadenopathy and reticular densities in both lung fields. Which of the following is a defining characteristic of the disorder suggested by these findings? (A)Abnormalities restricted to lung and hilar lymph nodes (B)Hypocalcemia (C)Impaired synthesis of immunoglobulins (D)Marked hyperreactivity to tuberculin (E)Noncaseating granuloma

The answer is C: Lymphangioleiomyomatosis. Lymphangio-leiomyomatosis is a rare interstitial lung disease that occurs in women of childbearing age. It is characterized by the wide-spread abnormal proliferation of smooth muscle in the lung (see photomicrograph), mediastinal and retroperitoneal lymph nodes, and the major lymphatic ducts. On gross examination, the lungs show bilateral, diffuse enlargement, with extensive cystic changes resembling those of emphysema. Hormonal ablation through oophorectomy and antiestrogen and proges-terone therapy has shown some promise. None of the other choices exhibit this morphologic pattern. Diagnosis: Lymphangioleiomyomatosis

A 30-year-old woman presents with shortness of breath and bloody sputum. Physical examination reveals pulmonary crackles and abdominal ascites. A chest X-ray shows bilat-eral pleural effusions and marked hyperinfl ation of the lungs. A CT scan of the chest discloses thin-walled, air-containing cysts in a diffuse symmetric pattern. A lung biopsy is shown in the image. The patient responds favorably to antiestrogen and antiprogesterone therapy. Which of the following is the most likely diagnosis? (A) Bronchiectasis (B) Histiocytosis X (C) Lymphangioleiomyomatosis (D) Tuberculosis (E) Wegener granulomatosis

The answer is B: Langerhans cell histiocytosis. Different pre-sentations of Langerhans cell histiocytosis have been called eosinophilic granuloma, Hand-Schuller-Christian disease, and Letterer-Siwe disease. In adults, the disorder occurs most often as an isolated form known as pulmonary eosinophilic granuloma. Virtually all of these patients are cigarette smok-ers. The pulmonary lesions consist of varying proportions of Langerhans cells admixed with lymphocytes, eosinophils, and macrophages. Eosinophils are not typical of the other choices. Diagnosis: Langerhans histiocytosis

A 31-year-old woman smoker complains of nonproduc-tive cough, chest pain, shortness of breath on exertion, and fatigue. A CBC is normal. A chest X-ray shows ill-defi ned nod-ules, reticulonodular infi ltrates, a small cavitary lesion in the right middle lobe, and mediastinal adenopathy. A transbron-chial biopsy is shown in the image. Which of the following is the most likely diagnosis? (A) Goodpasture syndrome (B) Langerhans cell histiocytosis (C) Lymphangioleiomyomatosis (D) Pulmonary interstitial fi brosis (E) Wegener granulomatosis

The answer is A: a1-Antitrypsin defi ciency. Hereditary defi -ciency of α1-antitrypsin accounts for about 1% of all patients with a clinical diagnosis of chronic obstructive pulmonary dis-ease and is considerably more common in young persons with severe emphysema. Emphysema in patients with this genetic disease is diffuse and is classifi ed as panacinar. In the lung, the most important action of α1-antitrypsin is its inhibition of neutrophil elastase, an enzyme that digests elastin and other structural components of the alveolar septa. Most patients with clinically-diagnosed emphysema who are younger than 40 years of age have α1-antitrypsin (PiZZ phenotype) defi -ciency. Emphysema is not a feature of the other choices. Diagnosis: α1-Antitrypsin defi ciency, emphysema

A 35-year-old woman with a long history of dyspnea, chronic cough, sputum production, and wheezing dies of respiratory failure following a bout of lobar pneumonia. She was a non-smoker and did not drink alcoholic beverages. The lung at autopsy is shown in the image. Which of the following under-lying conditions was most likely associated with the patho-logic changes shown here? (A) α1-Antitrypsin defi ciency (B) Cystic fi brosis (C) Goodpasture syndrome (D) Hypersensitivity pneumonitis (E) Kartagener syndrome

The answer is a. Leukocytoclastic angiitis refers to the histologic finding of fragmentedneutrophils surrounding small blood vessels. The differential diagnosis of leukocytoclastic vasculitis includes microscopic PAN and three other disorders: Henoch-Schönleinpurpura, Wegener granulomatosis, and Churg-Strauss syndrome. Henoch-Schönlein purpura is a disorder of children who present with hemorrhagic urticaria and hematuriafollowing an upper respiratory infection. The pathology of this disease involves the deposition of immunoglobulin A (IgA) immune complexes in small vessels of the skin.Because the antibody is IgA, the alternate complement pathway is activated in these patients. Wegener granulomatosis (WG) is characterized by acute necrotizing granulomasof the upper and lower respiratory tract, focal necrotizing vasculitis affecting small- to medium-sized vessels, and renal disease. Histologically, there is fibrinoid necrosis ofsmall arteries, early infiltration by neutrophils, and granuloma formation with giant cells. The peak incidence is in the fifth decade, and many patients have C-ANCAs. Thedisease is highly fatal, with de

A 38-year-old woman presents with new onset of multiple purpuric skin lesions. Two years ago she developed late-onset asthma and mild hypertension. Laboratoryexamination reveals an increase in the number of eosinophils in the peripheral blood (peripheral eosinophilia), and a biopsy from one of the purpuric skin lesions revealsleukocytoclastic vasculitis. No perivascular IgA deposits are found, and no antineutrophil cytoplasm autoantibodies are present. Which of the following is the most likelydiagnosis? a. Churg-Strauss syndrome b. Henoch-Schönlein purpura c. Macroscopic polyarteritis nodosa d. Microscopic polyangiitis e. Wegener granulomatosis

The answer is a. One type of bronchogenic carcinoma that has unique characteristics is bronchioloalveolarcarcinoma (BAC). This tumor is characterized by well-differentiated, mucus-secreting columnar epithelial cells that infiltrate along the alveolar walls and spread from alveolusto alveolus through the pores of Kohn. This pneumonic spread can be mistaken for pneumonia on chest x-ray. These tumors, which make up about 2% to 5% ofbronchogenic carcinomas, do not arise from the major bronchi. Instead they are thought to arise in terminal bronchioles from Clara cells. Even though these tumors may bemultiple, they are well differentiated and have a good prognosis

A 39-year-old woman presents with a cough and increasing shortness of breath. A chest x-ray is interpreted by the radiologist as showing a right lower lobe (RLL)pneumonia. No mass lesions are seen. The woman is treated with antibiotics, but her symptoms do not improve. On her return visit, the area of consolidation appears to beincreased. Bronchoscopy is performed. No bronchial masses are seen, but a transbronchial biopsy is obtained in an area of mucosal erythema in the RLL. After the diagnosis ismade, the RLL is removed and a section from this specimen reveals well-differentiated mucus-secreting columnar epithelial cells that infiltrate from alveolus to alveolus. Which of the following is the most likely diagnosis? a. Bronchioloalveolar carcinoma b. Carcinoid c. Large-cell carcinoma d. Small-cell carcinoma e. squamous cell carcinoma

The answer is D: Lobar pneumonia. The term lobar pneumo-nia refers to consolidation of an entire lobe; bronchopneu-monia (choice B) signifi es scattered solid foci in the same or several lobes. Lobar pneumonia presents with a diffuse con-solidation of one or more pulmonary lobes. In contrast to lobar pneumonia, interstitial pneumonia (choice C) primarily involves the alveolar septa. Atypical pneumonia (choice A) is most often encountered in mycoplasma pneumonia. Pulmo-nary abscess (choice E) may be a complication of lobar pneu-monia or bronchopneumonia. Diagnosis: Lobar pneumoni

A 40-year-old alcoholic man is admitted to the hospital in severe respiratory distress. The temperature is 38.7°C (103°F), respirations are 32 per minute, and blood pressure is 130/90 mm Hg. He coughs constantly and expectorates "currant-jelly" sputum. A chest X-ray reveals bilateral diffuse pulmonary consolidation. Physical examination shows bilat-eral crackles, dullness to percussion over both pulmonary fi elds, and use of accessory muscles. The patient subsequently dies from complications of bacterial sepsis. The left lung at autopsy (shown in the image) shows a red, engorged lower lobe. What is the appropriate diagnosis? (A) Atypical pneumonia (B) Bronchopneumonia (C) Interstitial pneumonia (D) Lobar pneumonia (E) Pulmonary abscess

The answer is E: Lymphocytic interstitial pneumonia. Lym-phocytic interstitial pneumonia (LIP) is a rare pneumonitis in which lymphoid infi ltrates are distributed diffusely in the interstitial spaces of the lung. In this case, the walls of the alveolar septa are diffusely infi ltrated with chronic infl am-matory cells. LIP often occurs in a variety of clinical settings, including Sjögren syndrome and HIV infection. The course of the disease varies from an indolent condition to one that progresses to end-stage lung disease and respiratory failure. Langerhans cell histiocytosis (choice C) features nodular infi l-trates. Interstitial lymphocytic infi ltrates are not characteristic of the other choices. Diagnosis: Lymphocytic interstitial pneumonia

A 43-year-old woman with Sjögren syndrome and a 5-year history of cough and shortness of breath develops end-stage lung disease and dies of respiratory failure. Histologic exami-nation of the lung at autopsy is shown in the image. Which of the following is the most likely diagnosis? (A) Alveolar proteinosis (B) Churg-Strauss syndrome (C) Langerhans cell histiocytosis (D) Lymphangioleiomyomatosis (E) Lymphocytic interstitial pneumonia

The answer is d. The pulmonary hemorrhagic syndromes are characterized by hemorrhage within the alveoli,which may be severe enough to produce hemoptysis. Several of these diseases are associated with blood vessel abnormalities, namely, inflammation of the vessels (angiitis).Necrotizing granulomatous arteritis affecting the upper and lower respiratory tracts and the kidneys is seen in patients with Wegener granulomatosis. These areas of necrosisare characteristically large and serpiginous, and exhibit peripheral palisading of macrophages. In patients with Wegener granulomatosis, the nose, sinus, antrum, and tracheaoften exhibit ulcerations. Originally the disease was lethal, but the prognosis is now much improved by immunosuppressive drugs. Laboratory findings reveal a positive ANCA(if kidney involved), mainly c-ANCA. Involvement of the kidneys may produce hematuria.Goodpasture syndrome is another pulmonary hemorrhagic syndrome that may produce hematuria. This syndrome is characterized by the development of a necrotizinghemorrhagic interstitial pneumonitis and rapidly progressing glomerulonephritis because of antibodies directed against the basement membrane

A 44-year-old woman presents with hemoptysis and hematuria. Pertinent medical history includes repeated infections involving her sinuses. Physical examination findsa small perforation of her nasal septum with several surrounding mucosal ulcers, and a small amount of purulent material is seen leaking from her sinuses. Serum studies arepositive for the presence of antineutrophil cytoplasmic antibodies, mainly against proteinase 3. Which of the following histologic abnormalities is most likely to be seen in alung biopsy from this individual? a. Atypical lymphocytes invading blood vessels b. Granulomatous inflammation of blood vessels with numerous eosinophils c. Granulomatous inflammation of bronchi with Aspergillus species d. Large, serpiginous necrosis with peripheral, palisading macrophages e. Necrotizing hemorrhagic interstitial pneumonitis

The answer is C: Large cell carcinoma. Large cell undifferenti-ated carcinoma is composed of atypical neoplastic cells that do not resemble any normal cells in the lung. These cells do not form glands (like adenocarcinoma) and do not express cytokeratin (choices A and D). Chromogranin is expressed in carcinoid tumors (choice B) and often in small cell carcinomas (choice E). Diagnosis: Large cell carcinoma of lung

A 67-year-old woman with a history of smoking presents with a 3-week history of chest pain and bloody sputum. A chest X-ray reveals a bulky mass within the pulmonary parenchyma. An open-lung biopsy is shown in the image. Immunohistochemical stains for keratin and chromogranin are negative. What is the appropriate diagnosis? (A) Adenocarcinoma (B) Carcinoid tumor (C) Large cell carcinoma (D) Metastatic adenocarcinoma (E) Small cell carcinoma

The answer is d. Pulmonary alveolar proteinosis (PAP) is a rare disease that is characterizedhistologically by the accumulation of intensely eosinophilic, proteinaceous, granular substance in the intra-alveolar and bronchiolar spaces. The alveolar walls are relativelynormal, without inflammatory exudate or fibrosis, although type II pneumocytes may be hyperplastic. The process is often patchy, with groups of normal alveoli alternatingwith groups of affected alveoli. Acicular (cholesterol) clefts and densely eosinophilic bodies (necrotic cells) are found within the granular material. Distinction from edemafluid may be difficult, but PAP alveolar material stains with periodic acid-Schiff (PAS).There are three distinct clinical classes of this disease: acquired, congenital, and secondary PAP. Currently it is postulated that acquired PAP is associated with thedevelopment of anti-GM-CSF antibodies. Congenital PAP is quite rare and is associated with mutations of genes coding for either surfactant protein B, GM-CSF, and GMreceptor β chain. Secondary PAP is associated with occupational exposure to silica or aluminum dusts.

A 45-year-old man presents with shortness of breath, cough with mucoid sputum, and some weight loss, and has diffuse, bilateral alveolar infiltrates on chest x-ray.Pulmonary function tests reveal decreased diffusing capacity and hypoxia. A lung biopsy reveals intensely eosinophilic granular material within alveoli. Workup finds thepresence of anti-GM-CSF antibodies in his serum. What is the best diagnosis? a. CREST syndrome b. Hypersensitivity pneumonitis c. Idiopathic pulmonary hemosiderosis d. Pulmonary alveolar proteinosis e. Respiratory bronchiolitis

The answer is a. Cryptogenic organizing pneumonia (COP) describes a characteristic histologic appearance ofpolypoid plugs of loose organizing connective tissue within alveolar ducts, alveoli, and bronchioles. This histologic appearance of loose (somewhat edematous) organizingfibrous tissue is called "organizing pneumonia." This same histologic appearance can also be seen as a response to several infections or inflammatory disorders of the lungs.The loose connective tissue, however, in COP is all of the same age, and the adjacent lung parenchyma is within normal limits. COP used to be called bronchiolitis obliterans-organizing pneumonia (BOOP). Patients usually present with symptoms of a flu-like illness, including cough, fever, malaise, fatigue, and weight loss. Chest x-rays revealsubpleural or peribronchial patchy areas of consolidation.In contrast to COP, interstitial or idiopathic pulmonary fibrosis (IPF) is a slowly progressive disease with no recognizable etiology. This disease entity has many names,such as chronic interstitial pneumonitis and cryptogenic fibrosing alveolitis. Another common name is usual interstitial pneumonitis (UIP)

A 45-year-old woman presents with a 3-week history of cough, fever, malaise, and increasing shortness of breath. A chest x-ray reveals widespread white patches, whilea transbronchial lung biopsy reveals polypoid plugs of loose fibrous tissue within bronchioles and alveoli. The adjacent lung tissue is unremarkable, and sheets of"desquamated" cells within alveoli are not seen. What is the best diagnosis? a. Cryptogenic organizing pneumonia (COP) b. Desquamative interstitial pneumonia (DIP) c. Idiopathic pulmonary fibrosis (IPF) d. Nonspecific interstitial pneumonia (NSIP) e. Primary atypical pneumonia (PAP

the answer is e. This woman has an adenocarcinoma. These are the most likely lung cancers to arise in never-smokers and are more common in women. They have been associated with EGFR mutations, and EGFR mutation testing is rapidly becoming the standard-of-care for lung adenocarcinomas in order to direct chemotherapy (EGFR-mutated cases may respond to targeted tyrosine kinase inhibitors). Adenocarcinomas may be preceded by or associated with atypical adenomatous hyperplasia, not squamous dysplasia. Unlike small cell carcinomas, they do not show neuroendocrine features such as synaptophysin staining and metastasis at the time of presentation is not the rule.

A 45-year-old woman with no smok-ing history presents with new onset of cough, shortness of breath, and weight loss. Imaging reveals a peripherally located lung mass with no evidence of primary tumor elsewhere in the body. On light microscopy, the tumor is comprised of poorly formed glands. Which of the following is most likely to be true of this tumor? (a)Positive for synaptophysin immunos-taining (B)Associated squamous dysplasia at the periphery of the tumor (c)Most likely already metastatic, therefore surgery is not recommended (D)Driven by tobacco-associated carcino-gens (e)Positive for mutation in EGFR

The answer is D: Pulmonary hypertension. Pulmonary hyper-tension is characterized by thickening of the media of pulmo-nary muscular arteries. As pulmonary hypertension becomes more severe, there is extensive intimal fi brosis and muscle thickening within arteries and arterioles, which may be occlu-sive. Churg-Strauss syndrome (choice A) exhibits vasculitis and eosinophilia but is excluded in this case on the basis of a normal CBC. The other choices do not principally affect arteries. Diagnosis: Pulmonary hypertension

A 45-year-old woman with severe kyphoscoliosis presents with fatigue, shortness of breath on exertion, fainting spells, and bloody sputum. A CBC is normal. Physical examination shows splitting of the second heart sound with accentuation of the pulmonic component, distended neck veins with aprominent V wave, a right ventricular third heart sound, and peripheral edema. A chest X-ray fi lm shows enlargement of the central pulmonary arteries. A transbronchial lung biopsy is shown in the image. Which of the following is the most likely diagnosis? (A) Churg-Strauss syndrome (B) Diffuse alveolar damage (C) Eosinophilic granuloma (D) Pulmonary hypertension (E) Wegener granulomatosis

he answer is B: Chronic obstructive pulmonary disease. Chronic obstructive pulmonary disease is a nonspecifi c term that describes patients with chronic bronchitis, emphysema, or both who evidence obstruction to air fl ow in the lungs. It is often diffi cult to separate the relative contribution of each disease to the clinical presentation. Chronic bronchitis is defi ned clinically as the presence of a chronic productive cough without a discernible cause for more than half the time over a period of 2 years. It is primarily a disease of cigarette smoking, with 90% of all cases occurring in smokers. The frequency and severity of acute respiratory tract infections is increased in patients with chronic bronchitis. Exertional dys-pnea and cyanosis supervene, and cor pulmonale may ensue. The combination of cyanosis and edema secondary to cor pul-monale has led to the label "blue bloater" for such patients. In contrast to patients with predominantly chronic bronchitis, those with emphysema are at lower risk of recurrent pulmo-nary infections and are not so prone to the development of cor pulmonale. The clinical course of emphysema is marked by an inexorable decline in respiratory function and

A 48-year-old man with a history of heavy smoking presents with a 3-year history of persistent cough and frequent upper respiratory infections, associated with sputum production. Physical examination reveals prominent expiratory wheezes and peripheral edema. Analysis of arterial blood gases reveals hypoxia and CO2 retention. Which of the following is the appropriate diagnosis? (A) Atelectasis (B) Chronic obstructive pulmonary disease (C) Goodpasture syndrome (D) Hypersensitivity pneumonitis (E) Usual interstitial pneumonia

The answer is E: Metastatic carcinoma. Pulmonary metasta-ses represent the most common neoplasm of the lung. In one third of all fatal cancers, pulmonary metastases are evident at autopsy. Metastatic carcinomas typically present as multiple, round masses scattered at random throughout the paren-chyma of lungs and liver. Although pulmonary adenocarci-noma (choice A) and bronchoalveolar carcinoma (choice B) cannot be excluded on histologic grounds, this patient with ulcerative colitis is predisposed to develop adenocarcinoma of the colon, which most likely accounts for the anemia and lung metastases. Diagnosis: Metastatic carcinoma of lung

A 48-year-old woman with a long-standing history of ulcer-ative colitis presents with anemia and shortness of breath. Laboratory studies show increased serum levels of carcinoem-bryonic antigen. A chest X-ray reveals multiple, round masses in both lungs. Histologic examination of an open-lung biopsy discloses nodules that are composed of gland-like structures. What is the most likely diagnosis? (A) Adenocarcinoma (B) Bronchioloalveolar carcinoma (C) Eosinophilic granuloma (D) Large cell undifferentiated carcinoma (E) Metastatic carcinoma

The answer is E. Small cell carcinoma of the lung is almost always metastatic at the time of initial diagnosis and is thus poorly amenable to surgery. Despite morphologic differ-ences, it is thought that all lung carcinomas, including small cell carcinoma, share a com-mon endodermal origin. The location is most often central rather than peripheral, and there is a marked association with cigarette smoking. Paraneoplastic syndromes include inappropriate secretion of ACTH and ADH. Secretion of a protein with PTH-like activity is an association of squamous cell lung carcinoma.

A 49-year-old man has a recent diagnosis of small cell carcinoma of the lung. Which of the following is an important characteristic of this form of lung cancer? (A)Ectodermal origin (B)Frequent peripheral location (C)Less association with cigarette smokingthan other forms of lung cancer (D)Paraneoplastic hyperparathyroidism (E)Poorly amenable to surgery

the answer is D. This patient has UIp, which carries a dismal prognosis characterized by refractoriness to steroids and development of honeycomb lung resulting in death, often within 5 years of diagnosis. The key histologic feature is temporal heterogeneity in the fibrotic changes, whereas LIP shows chronologically uniform, diffuse fibrosis. In contrast to DIP, UIP is not related to smoking. It is not attributed to any environmental stimulus or pathogen

A 50-year-old female presents with restrictive lung disease. She describes an aggressive clinical course with rapidly pro-gressive shortness of breath over the last year. A lung biopsy reveals a patchy process characterized by temporally heterogeneous areas of fibrosis. Which of the following is most likely of her expected clinical course? (a)Symptoms should abate with smoking cessation and steroid treatment (B)Prognosis is relatively good with the majority of patients surviving at 10 years (c)Excellent prognosis following removal of environmental stimulus (D)Poor prognosis with development of honeycomb lung and death within 5years (e)Symptoms should improve with antibi-otic therapy

the answer is a. Bronchial asthma, or hyperreactive airway disease, is a type of COPD caused by narrowing of airways. Asthma manifests morphologically by bronchial smooth muscle hypertrophy, hyperplasia of bronchial submucosal glands and goblet cells, and airways plugged by mucus-containing Curschmann spirals (whorl-like accumulations of epithelial cells), eosinophils, and Charcot-Leyden crystals (crystalloids of eosinophil-derived proteins).

A 50-year-old man dies of a respiratory illness that had been characterized by dys-pnea, cough, and wheezing expiration of many years' duration. Initially episodic, his "attacks" had increased in frequency and at the time of death had become continuous and intractable. At autopsy, which of the fol-lowing is the most likely histologic finding in the lungs? (a)Bronchial smooth muscle hypertrophy with proliferation of eosinophils (B)Diffuse alveolar damage with leakage of protein-rich fluid into alveolar spaces (c)Dilation of air spaces with destruction of alveolar walls (D)Hyperplasia of bronchial mucus-secreting submucosal glands (e)Permanent bronchial dilation caused by chronic infection, with bronchi filled with mucus and neutrophils

The answer is C: Hypersensitivity pneumonitis. Hypersensi-tivity pneumonitis (extrinsic allergic alveolitis) is a response to inhaled antigens. The inhalation of a variety of antigens leads to acute or chronic interstitial infl ammation in the lung. Hypersensitivity pneumonitis may develop in response to repeated exposure to a variety of organic materials (e.g., bird droppings, feathers, mushrooms, and tree bark). Histo-logically, the lung contains poorly formed granulomas, which differ from the compact (solid) noncaseating granulomas of sarcoidosis and the caseating granulomas of tuberculosis or histoplasmosis. Actinomycosis and nocardiosis (choices A and D) do not induce granulomas. Wegener granulomatosis (choice E) is not known to be associated with environmental exposure. Diagnosis: Hypersensitivity pneumonitis, pigeon breeder lung disease

A 50-year-old woman presents with a 4-week history of fever, shortness of breath, and dry cough. She reports that her chest feels "tight." The patient is a pigeon fancier. Blood tests show leukocytosis and neutrophilia, an elevated erythrocyte sedimentation rate, and increased levels of immunoglobulins and C-reactive protein. A lung biopsy reveals poorly formed granulomas composed of epithelioid macrophages and multi-nucleated giant cells. Which of the following is the appropri-ate diagnosis? (A) Actinomycosis (B) Goodpasture syndrome (C) Hypersensitivity pneumonitis (D) Nocardiosis (E) Wegener granulomatosis

The answer is E: Small cell carcinoma. Small cell carcinoma (previously referred to as "oat-cell" carcinoma) is a highly malignant epithelial tumor of the lung that exhibits neuroen-docrine features. It accounts for 20% of all lung cancers and is strongly associated with cigarette smoking. Metastases occur early and are widespread. Carcinoid tumors (choice C) also contain neuroendocrine granules, but the tumor cells are arranged in a distinctive pattern. Moreover, Cushing syndrome is often encountered in patients with small cell carcinoma, but not carcinoid tumor (choice C). Diagnosis: Small cell carcinoma of lung

A 52-year-old woman presents with a 1-year history of upper truncal obesity and moderate depression. Physical examina-tion shows hirsutism and moon facies. Endocrinologic stud-ies reveal hypokalemia, high plasma corticotropin levels, and increased concentrations of serum and urine cortisol. CT scan of the thorax demonstrates a hilar mass. A transbronchial lung biopsy is shown in the image. Electron microscopy discloses neuroendocrine granules within the cytoplasm of some tumor cells. What is the appropriate diagnosis? (A) Adenocarcinoma (B) Bronchioloalveolar carcinoma (C) Carcinoid tumor (D) Metastatic carcinoma (E) Small cell carcinoma

The answer is E: Neuroendocrine granules. Neuroendocrine tumors may synthesize a number of hormones. The presence of small, membrane-bound granules with a dense core is a fea-ture of these neoplasms. Dense granules are visible by electron microscopy. In this way, electron microscopy may aid in the diagnosis of poorly differentiated cancers, whose classifi cation is problematic by light microscopy. Carcinomas often exhibit desmosomes and specialized junctional complexes, which are structures that are not typical of sarcomas or lymphomas. Myelin fi gures (choice D) are seen in patients with inherited lysosomal storage disease. Councilman bodies (choice A) are apoptotic hepatocytes (acidophilic bodies). Diagnosis: Small cell carcinoma of lung, paraneoplastic syndrom

A 52-year-old woman presents with a 1-year history of upper truncal obesity and moderate depression. Physical examination shows hirsutism and moon facies. A CT scan of the thorax dis-plays a hilar mass. A transbronchial lung biopsy discloses small cell carcinoma. Electron microscopy of this patient's lung tumor will most likely reveal which of the following cytologic features? (A) Councilman bodies (B) Hyperplasia of endoplasmic reticulum (C) Mitochondrial calcifi cation (D) Myelin fi gures in lysosomes (E) Neuroendocrine granules

The answer is D: Usual interstitial pneumonia (UIP). UIP is one of the most common types of interstitial pneumonia and demonstrates a histologic pattern that occurs in a variety of clinical settings, including collagen vascular disease, chronic hypersensitivity pneumonitis, and drug toxicity. Most com-monly, it has no known cause, although viral, genetic, and immunologic factors may be implicated. A microscopic view of the lung in this case shows patchy, subpleural fi brosis with microscopic "honeycomb" cystic change. Diffuse fi brosis is not characteristic of the other choices. Diagnosis: Usual interstitial pneumonia

A 53-year-old man presents with increasing shortness of breath on exertion and dry cough that has developed over a period of a few years. Physical examination shows clubbing of the fi ngers. A chest X-ray discloses diffuse bilateral infi ltrates, predominantly in the lower lobes, in a reticular pattern. Two years later, the patient suffers a massive stroke and expires. Histologic examination of the lung at autopsy is shown in the image. Patchy scarring with extensive areas of honeycomb cystic change predominantly affects the lower lobes. Which of the following is the most likely diagnosis? A) Churg-Strauss syndrome (B) Desquamative interstitial pneumonia (C) Goodpasture syndrome (D) Usual interstitial pneumonia (E) Wegener granulomatosis

The answer is A: Adenocarcinoma. Adenocarcinoma usually presents as a peripheral subpleural mass composed of neo-plastic gland-like structures. Central (hilar) cancers of the lung can be of any of the histologic types (e.g., choices B, D, and E), whereas peripheral lung cancers are most com-monly diagnosed as adenocarcinomas. They are often asso-ciated with pleural fi brosis and subpleural scars. At initial presentation, adenocarcinomas usually appear as irregular masses, although they may be so large that they completely replace the entire lobe of the lung. Mesothelioma (choice C) is pleural based. Diagnosis: Adenocarcinoma of lung

A 53-year-old woman with a history of cigarette smoking pres-ents with a 3-month history of chest pain, cough, and mild fever. A chest X-ray reveals a peripheral mass in the left upper lobe. The surgical specimen is shown in the image. What is the most likely diagnosis? A) Adenocarcinoma (B) Large cell carcinoma (C) Mesothelioma (D) Small cell carcinoma (E) Squamous cell carcinoma

The answer is C: Metastatic carcinoma of the lung. This patient's lung shows numerous nodules of metastatic car-cinoma corresponding to "cannon ball" metastases seen radiologically. Pulmonary metastases are more common than primary lung tumors, and the histologic appearance of most metastases resembles that of the primary tumor. Persons with ulcerative colitis (such as this patient) have a higher risk of colorectal cancer than the general population. The risk is related to the extent of colorectal involvement and the dura-tion of the infl ammatory disease. Carcinoid tumor of the lung (choice A) and primary lung cancer (choice B) would not typi-cally show multiple, circumscribed nodules. Miliary tuberculosis (choice D) and sarcoidosis (choice E) feature mm-sized infl ammatory nodules (minute granulomas). Diagnosis: Metastatic cancer, metastatic carcinoma of the lung

A 53-year-old woman with a longstanding history of ulcerative colitis presents with increasing chest pain and shortness of breath of 2 months duration. She reports four recent epi-sodes of hemoptysis. The patient subsequently develops over-whelming sepsis and expires. A section through the right lung is examined at autopsy (shown in the image). What is the appropriate diagnosis? (A) Carcinoid tumor of the lung (B) Primary adenocarcinoma of the lung (C) Metastatic carcinoma of the lung (D) Miliary tuberculosis (E) Sarcoidosis

The answer is d. Horner syndrome occurs with apical (superior sulcus) tumors of any type (Pancoast tumor), but because mostperipheral cancers of the lung are adenocarcinomas, most tumors of the apex of the lung are adenocarcinomas. Horner syndrome is characterized by enophthalmos, ptosis,miosis, and anhidrosis on the same side as the lesion due to invasion of the cervical sympathetic nerves. Involvement of the brachial plexus causes pain and paralysis in theulnar nerve distribution.

A 54-year-old man presents with several problems involving his face and pain in his shoulder. He states that he has smoked 2 packs of cigarettes a day for almost 40years. Physical examination reveals ptosis of his left upper eyelid, constriction of his left pupil, and lack of sweating (anhidrosis) on the left side of his face. No otherneurologic abnormalities are found. Which of the following tumors is most likely to be present in this individual? a. A bronchioloalveolar carcinoma involving the left upper lobe b. A small-cell carcinoma involving the hilum of his left lung c. A squamous cell carcinoma involving the left mainstem bronchus d. An adenocarcinoma involving the apex of his left lung e. An endobronchial carcinoid tumor involving the right mainstem bronchus

The answer is C: Chronic vascular rejection. Chronic vascular rejection, also referred to as accelerated coronary artery disease, is the most common cause of death in heart transplant patients after the fi rst year of transplantation. It usually affects the proximal and distal coronary arteries and their penetrat-ing branches. Microscopically, the disorder is characterized by concentric intimal proliferation, leading to occlusion and myocardial infarction. This complication is painless because the transplanted heart is denervated. Acute cellular graft rejection (choice A) occurs during the fi rst few months after trans-plantation, and hyperacute graft rejection (choice D) occurs quickly after transplantation. Diagnosis: Coronary artery disease

A 55-year-old man with idiopathic dilated cardiomyopathy receives a heart transplant. Which of the following is the most likely cause of death in this patient 2 years after transplantation? (A) Acute cellular graft rejection (B) Aortic valve stenosis (C) Chronic vascular rejection (D) Hyperacute graft rejection (E) Pulmonary fibrosis

The answer is A: Destruction of the walls of airspaces without fi brosis. Emphysema is a chronic lung disease characterized by enlargement of the airspaces distal to the terminal bronchi-oles, with destruction of their walls, but without fi brosis or infl ammation. Diagnosis: Emphysema

A 55-year-old man is admitted to the hospital with increasing shortness of breath and dry cough for the past few years. He smokes 1.5 packs of cigarettes and drinks about four bottles of beer a day. He is constantly "gasping for air" and now walks with diffi culty because he becomes breathless after only a few steps. Prolonged expiration with wheezing is noted. Physical examination shows a barrel chest, hyperresonance on percus-sion, and clubbing of the digits. The patient's face is puffy and red, and he has pitting edema of the legs. A chest X-ray discloses hyperinfl ation, fl attening of the diaphragm, and increased retrosternal air space. Which of the following best describes the expected histopathology of the lungs in the patient described above? (A) Destruction of the walls of airspaces without fi brosis (B) Interstitial fi brosis of the lung parenchyma (C) Lymphocytes restricted to the interstitium (D) Prominent bronchial smooth muscle cell hyperplasia (E) Thickening of the epithelial basement membrane

he answer is C: Emphysema. Chronic obstructive pulmo-nary disease is a nonspecifi c term that describes patients with chronic bronchitis or emphysema who evidence a decrease in forced expiratory volume. Emphysema is characterized prin-cipally by hyperinfl ated lungs. Chronic bronchitis (choice B) occurs after recurrent infections and, like asthma (choice A), is not generally associated with hyperinfl ated lungs. The major cause of emphysema is cigarette smoking, and moderate-to-severe emphysema is rare in nonsmokers .Diagnosis: Emphysem

A 55-year-old man is admitted to the hospital with increasing shortness of breath and dry cough for the past few years. He smokes 1.5 packs of cigarettes and drinks about four bottles of beer a day. He is constantly "gasping for air" and now walks with diffi culty because he becomes breathless after only a few steps. Prolonged expiration with wheezing is noted. Physical examination shows a barrel chest, hyperresonance on percus-sion, and clubbing of the digits. The patient's face is puffy and red, and he has pitting edema of the legs. A chest X-ray discloses hyperinfl ation, fl attening of the diaphragm, and increased retrosternal air space. Which of the following is the appropriate diagnosis? (A) Asthma (B) Chronic bronchitis (C) Emphysema (D) Hypersensitivity pneumonitis (E) Usual interstitial pneumonia

The answer is C: Carcinoid tumor. Carcinoid tumors account for 2% of all primary lung cancers. They comprise a group of neuroendocrine neoplasms derived from the pluripotential basal layer of the respiratory epithelium. Carcinoid tumors occur most often in the wall of the major bronchus and may protrude into its lumen. The tumors are characterized by an organoid growth pattern and uniform cytologic features. Carcinoid tumors exhibit a neuroendocrine differentiation similar to that of resident Kulchitsky cells. The indolent nature of carcinoid tumors is refl ected in the fi nding that half of the patients are asymptomatic at the time of presen-tation, but regional lymph node metastases occur in 20% of patients. Atypical carcinoids exhibit a more aggressive behavior. Neuroendocrine features are absent in the other tumors. Diagnosis: Carcinoid tumor of lung

A 55-year-old man presents with increasing chest pain, bloody sputum, and weight loss over the past 3 months. A high-resolution CT scan reveals a mass circumscribing the right main bronchus, extending into its lumen. Histologic examina-tion of an open-lung biopsy is shown in the image. Electron microscopy shows numerous neuroendocrine granules within tumor cells. What is the appropriate diagnosis? (A) Adenocarcinoma (B) Bronchioloalveolar carcinoma (C) Carcinoid tumor (D) Large cell carcinoma (E) Squamous cell carcinoma

The answer is D: Elastase. The dominant hypoth-esis concerning the pathogenesis of emphysema is the proteolysis-antiproteolysis theory. In most patients with emphysema, it is thought that tobacco smoke induces an infl ammatory reaction. Serine elastase in neutrophils is a par-ticularly potent elastolytic agent, which injures the elastic tis-sue of the lung. Over time, an imbalance in elastin generation and catabolism in the lung leads to emphysema (i.e., emphy-sema results when elastolytic activity is increased or antielas-tolytic activity is reduced). α1-Antitrypsin (choice B), a circu-lating glycoprotein produced in the liver, is a major inhibitor of a variety of proteases, including elastase, and accounts for 90% of antiproteinase activity in the blood. Diagnosis: Emphysema

A 55-year-old man was admitted to the hospital with a chief com-plaint of increasing shortness of breath over the past several years. The patient was a heavy smoker over the past 40 years. Physical examination reveals cyanosis, elevated jugular venous pressure, and peripheral edema. A high-resolution CT scan shows bullae over both lungs. Chronic intra-alveolar exposure to which of the following proteins is most likely associated with the pathogenesis of chronic obstructive pulmonary disease in this patient? (A) Alkaline phosphatase (B) α1-Antitrypsin (C) Collagenase (D) Elastase (E) α2-Macroglobulin

The answer is C: Chronic vascular rejection. Chronic vascular rejection, also referred to as accelerated coronary artery dis-ease, is the most common cause of death in heart transplant patients after the fi rst year of transplantation. It usually affects the proximal and distal coronary arteries and their penetrat-ing branches. Microscopically, the disorder is characterized by concentric intimal proliferation, leading to occlusion and myocardial infarction. This complication is painless because the transplanted heart is denervated. Acute cellular graft rejec-tion (choice A) occurs during the fi rst few months after trans-plantation, and hyperacute graft rejection (choice D) occurs quickly after transplantation. Diagnosis: Coronary artery disease

A 55-year-old man with idiopathic dilated cardiomyopathy receives a heart transplant. Which of the following is the most likely cause of death in this patient 2 years after transplanta-tion? (A) Acute cellular graft rejection (B) Aortic valve stenosis (C) Chronic vascular rejection (D) Hyperacute graft rejection (E) Pulmonary fi brosis

The answer is D. Chronic bronchitis, which is clearly linked to cigarette smoking, is defined as productive cough occurring during at least 3 consecutive months over at least 2 consecutive years.

A 67-year-old man, a two-pack-a-day smoker since age 18, has had a produc-tive cough over the past 20 years. Although continuous through the years, there have been episodic exacerbations of these symp-toms, which have worsened during the past 4 or 5 years, lasting 3 or 4 months at a time. Arterial pO2 is decreased and pCO2is increased. Total lung capacity measure-ments are normal. These findings are most suggestive of which of the following pulmo-nary disorders? (A)Adult respiratory distress syndrome (ARDS) (B)Bronchial asthma (C)Lung carcinoma (D)Chronic bronchitis (E)Panacinar emphysema

The answer is C: Cryptogenic organizing pneumonia. Orga-nizing pneumonia was previously referred to as bronchiolitis obliterans-organizing pneumonia. The histologic pattern is not specifi c for any particular etiologic agent and may be observed in many settings, including respiratory tract infections, inhala-tion of toxic materials, and collagen vascular diseases. In the absence of a specifi c etiology, the term cryptogenic organizing pneumonia is applied. Loose fi brous tissue in the alveoli and bronchioles is a typical fi nding in patients with cryptogenic organizing pneumonia. Diffuse alveolar damage (choice D) features intra-alveolar fi brin (hyaline membranes). Diagnosis: Cryptogenic organizing pneumonia

A 55-year-old woman complains of sudden onset of fever, dry cough, and shortness of breath. She was seen for fl u-like symptoms 6 weeks ago. A chest X-ray shows bilateral patchy alveolar consolidations. An open-lung biopsy reveals narrow infl amed airways containing plugs of fi brous tissue (shown in the image). Which of the following is the most likely diagnosis? (A) AIDS-related pneumonia (B) Alveolar proteinosis (C) Cryptogenic organizing pneumonia (D) Diffuse alveolar damage (E) Wegener granulomatosis

he answer is E: Pulmonary hamartoma. Although the term hamartoma implies a malformation, hamartomas are true tumors. They are composed of cartilage, fi bromyxoid connective tissue, fat, bone, and occasional smooth mus-cle. They typically occur in adults, with a peak in the sixth decade of life. Hamartomas are the cause of approximately 10% of "coin" lesions discovered incidentally on chest radio-graphs. A characteristic "popcorn" pattern of calcifi cation is often seen by X-ray. Cartilage is not encountered in the other choices. Diagnosis: Pulmonary hamartom

A 56-year-old man undergoes a routine chest radiograph as part of a comprehensive physical examination. The X-ray fi lm of the chest shows a solitary, centrally located coin lesion, with a "popcorn" pattern of calcifi cation. A lung biopsy is per-formed and reveals nodules of benign mature cartilage and respiratory epithelium (shown in the image). What is the most likely diagnosis? (A) Carcinoid tumor (B) Extralobar sequestration (C) Leiomyoma (D) Pulmonary fi broma (E) Pulmonary hamartoma

The answer is D: Squamous cell carcinoma. The vast majority of laryngeal cancers are squamous cell carcinomas and occur principally in smokers. Adenocarcinoma (choice A), leiomyo-sarcoma (choice B), and small cell carcinoma (choice C) are rarely encountered in the larynx. Diagnosis: Laryngeal cancer

A 56-year-old man with a history of cigarette smoking pres-ents with diffi culty swallowing and a muffl ed voice. Laryngos-copy reveals a 2-cm laryngeal mass. If this mass is a malignant neoplasm, which of the following is the most likely histologic diagnosis? (A) Adenocarcinoma (B) Leiomyosarcoma (C) Small cell carcinoma (D) Squamous cell carcinoma (E) Transitional cell carcinoma

The answer is A: Bronchioloalveolar carcinoma. Bronchioloal-veolar carcinoma is a primary pulmonary adenocarcinoma originating from stem cells in the terminal bronchioles. The cells may be columnar and mucus producing or cuboidal and similar to type II pneumocytes. They tend to grow along the alveolar septa, as depicted in the photomicrograph. A similar growth pattern may be seen in metastatic adenocarcinomas. None of the other tumors produce alveolar mucus or display alveolar spaces lined by a columnar epithelium. Diagnosis: Bronchioloalveolar carcinoma

A 58-year-old man presents with a long history of persis-tent cough, chest pain, and recurrent pneumonia. He denies smoking or consuming alcohol. The patient subsequently dies of sepsis. Autopsy reveals malignant cells that diffusely infi ltrate the lung parenchyma. Histopathologic examina-tion of the lung shows well-differentiated, mucus-producing, columnar neoplastic cells lining the alveolar spaces (shown in the image). Neoplastic cells are not found in any other organ. What is the most likely diagnosis? (A) Bronchioloalveolar carcinoma (B) Carcinoid tumor (C) Large cell carcinoma (D) Mesothelioma (E) Small cell carcinoma

the answer is e. The man is likely to have a lung tumor, given his clinical presentation and the radiographic results. The patient's hypercalcemia is likely due to a paraneoplastic syndrome, such as that due to the elaboration of parathyroid-related hormone (PTrH). PTrH is produced by squamous cell carcinoma, whereas adrenocorticotropic-like sub-stance and antidiuretic hormone are produced by yet another form of lung cancer—small cell carcinoma of the lung. Carcinoembryonic antigen is an oncofetal antigen produced by colon cancer cells. Erythropoietin causes secondary polycythemia and is related to renal cell carcinoma.

A 58-year-old man with a 700-pack-per-year smoking history presents to the emer-gency department with shortness of breath and hemoptysis. Portable chest radiography demonstrates a large mass centrally located within the left lung field. The serum calcium is 13.0 mg/dL (normal 8.5 to 10.2). The met-abolic abnormality described here is likely due to elaboration of which substance? (a)Adrenocorticotropic hormone-like substance (B)Antidiuretic hormone (C)Carcinoembryonic antigen (d)Erythropoietin (e)Parathyroid-related hormone

The answer is C: Invasion of the underlying basement membrane. The fi rst event in tumor cell invasion is breach of the basement membrane that separates an epithelium from the underlying mesenchyme. After invading the interstitial tis-sue, malignant cells penetrate lymphatic or vascular channels (choice D). In the lymph nodes, communications between the lymphatics and venous tributaries allow malignant cells access to the systemic circulation. The other choices are important for tumor metastases, but they occur later than basement membrane invasion. Diagnosis: Adenocarcinoma of colon

A 58-year-old woman with colon cancer presents with 3 months of increasing shortness of breath. A chest X-ray reveals numerous, bilateral, round masses in both lungs. His-tologic examination of an open-lung biopsy discloses malig-nant gland-like structures, which are nearly identical to the colon primary. Which of the following changes in cell behav-ior was the fi rst step in the process leading to tumor metastasis from the colon to the lung in this patient? (A) Arrest within the circulating blood or lymph (B) Exit from the circulation into a new tissue (C) Invasion of the underlying basement membrane (D) Penetration of vascular or lymphatic channels (E) Stimulation of angiogenesis within the pulmonary metastases

The answer is e. Malignant mesothelioma arises from the pleural surfaces and develops with significant and chronicexposure to asbestos (usually occupationally incurred). As the malignant mesothelioma spreads, it lines the pleural surfaces, including the fissures through the lobes of thelungs, and results in a tight and constricting encasement. Adenocarcinoma of the lung also may invade the pleural surfaces and spread in an advancing manner throughout thepleural lining. The differential diagnosis grossly and also histologically between an epithelial type of malignant mesothelioma and an adenocarcinoma may be difficult.A characteristic feature seen by electron microscopy is numerous long microvilli on the surface of cells from mesotheliomas. In contrast, adenocarcinomas have short,plump microvilli. Other histologic characteristics that favor the diagnosis of malignant mesothelioma over adenocarcinoma include positive acid mucopolysaccharide stainingthat is inhibited by hyaluronidase, perinuclear keratin staining (not peripheral), positive staining for calretinin, and negative staining with carcinoembryonic antigen (CEA)and Leu-M1.

A 59-year-old man presents with marked weight loss and lethargy. Workup finds a diffuse mass filling the pleural space on the left side of his chest. Surgery is performedand gross examination finds a lesion encasing a large portion of the lung. Which one of the following features does not favor the diagnosis of malignant mesothelioma overadenocarcinoma? a. Numerous long microvilli seen with electron microscopy b. Perinuclear keratin staining c. Positive acid mucopolysaccharide staining that is inhibited by hyaluronidase d. Positive staining for calretinin e. Positive staining with carcinoembryonic antigen

The answer is D: Cytokeratins. Tumor markers are products of malignant neoplasms that can be detected in cells or body fl uids. Useful tumor markers include immunoglobulins, fetal proteins, enzymes, hormones, and cytoskeletal proteins. Car-cinomas uniformly express cytokeratins, which are interme-diate fi laments. Alpha-fetoprotein (choice A) is a marker for yolk sac carcinoma and hepatocellular carcinoma. Calretinin (choice B) provides a marker for mesothelioma. Carcinoem-bryonic antigen (choice C) is a marker for colon carcinoma and many other malignancies. Synaptophysin (choice E) is a marker for neuroendocrine tumors, including small cell carci-noma of the lung. Diagnosis: Squamous cell carcinoma of lung

A 60-year-old man presents with a 4-month history of increas-ing weight loss, wheezing, and shortness of breath. He has smoked two packs of cigarettes a day for 40 years. His past medical history is signifi cant for emphysema and chronic bronchitis. A chest X-ray shows a 10-cm mass in the left lung. Bronchoscopy discloses obstruction of the left main stem bronchus. A biopsy is obtained (shown in the image). Immu-nohistochemical studies of this biopsy specimen would most likely show strong expression of which of the following tumor markers? A) Alpha-fetoprotein (B) Calretinin (C) Carcinoembryonic antigen (D) Cytokeratins (E) Synaptophysin

the answer is D: Polycyclic aromatic hydrocarbons. Polycyclic aromatic hydrocarbons, originally derived from coal tar, are among the most extensively studied carcinogens. These com-pounds produce cancers at the site of application. Since poly-cyclic hydrocarbons have been identifi ed in cigarette smoke, it has been suggested (but not proved) that they are involved in the pathogenesis of lung cancer. Afl atoxin B1 (choice A), a natural product of the fungus Aspergillus fl avus, is among the most potent liver carcinogens. Asbestos (choice B), a mineral, is associated with mesothelioma and adenocarcinoma of lung. Industrial workers exposed to high levels of vinyl chloride (choice E) in the ambient atmosphere developed angiosarco-mas of the liver. Diagnosis: Squamous cell carcinoma of lu

A 60-year-old man presents with a 4-month history of increas-ing weight loss, wheezing, and shortness of breath. He has smoked two packs of cigarettes a day for 40 years. His past medical history is signifi cant for emphysema and chronic bronchitis. A chest X-ray shows a 10-cm mass in the left lung. Bronchoscopy discloses obstruction of the left main stem bronchus. A biopsy is obtained (shown in the image). Which of the following potent carcinogens was most likely involved in the pathogenesis of lung cancer in the patient described above? (A) Afl atoxin B1 (B) Asbestos (C) Azo dyes (D) Polycyclic aromatic hydrocarbons (E) Vinyl chloride

the answer is e. Asbestosis is caused by inhalation of asbestos fibers, characterized by yellow-brown, rod-shaped ferruginous bodies with clubbed ends that stain positively with Prussian blue. Asbestosis results in a marked predisposition to malignant mesothelioma of the pleura or peritoneum. Exposure to asbestos is also a risk factor for primary lung carci-noma, as well as for carcinoma of the oropharynx, esophagus, and colon. The risk of pri-mary lung carcinoma is greatly increased in cigarette smokers with exposure to asbestos.

A 60-year-old man presents with dys-pnea on exertion and a nonproductive cough. He has never smoked, but he worked as a shipbuilder, with known asbestos expo-sure approximately 20 years ago. To which of the following conditions is this patient especially predisposed? (a)Acute respiratory distress syndrome (B)Goodpasture syndrome (c)Idiopathic pulmonary fibrosis (D)Idiopathic pulmonary hemosiderosis (e)Malignant mesothelioma of the pleura

The answer is C. Exposure to asbestos (common in construction workers, shipyard work-ers, or people who have worked with insulation or fire safety materials) markedly pre-disposes one to mesothelioma of the pleura or peritoneum and is also closely linked to primary lung carcinoma (especially in smokers)

A 60-year-old man presents with the new onset of dyspnea, chest pain, cough, and weight loss. In the past, he had worked in construction, installing insulation in buildings. Chest radiograph shows a right-sided pleural effusion with marked pleural thickening and mass formation. A malignant neoplasm is demonstrated by biopsy. What is the most likely diagnosis? (A)Small cell carcinoma (B)Squamous cell carcinoma (C)Mesothelioma (D)Adenocarcinoma (E)Carcinoid tumor

the answer is c. Emphysema is an example of COPD. Due to the destruction of alveolar walls, a lack of elastic recoil causes air to become trapped in alveoli, and, thus, airflow obstruction occurs on expiration. In COPD, FEV1 is decreased, whereas FVC is normal or increased; therefore, patients with COPD have a decreased FEV1:FVC ratio

A 60-year-old man, a heavy smoker, pres-ents for advice to stop smoking. On physi-cal examination, he is thin and has a ruddy complexion. He has a productive cough and a barrel-shaped chest. He sits leaning forward with his lips pursed to facilitate his breathing. A diagnosis of emphysema is made. Which of the following is the most likely histologic finding in the lungs? (a)Bronchial smooth muscle hypertrophy with proliferation of eosinophils (B)Diffuse alveolar damage with leakage of protein-rich fluid into alveolar spaces (c)Dilation of air spaces with destruction of alveolar walls (D)Hyperplasia of bronchial mucus-secret-ing submucosal glands (e)Permanent bronchial dilation caused by chronic infection, with bronchi filled with mucus and neutrophils

The answer is D: Silicosis. Silicosis is caused by inhalation of small crystals of quartz (silicon dioxide), which are generated by stone cutting, sandblasting, and mining. The condition is marked by the insidious development of fi brotic pulmo-nary nodules containing quartz crystals. The disease may be asymptomatic for prolonged periods of time or may cause only mild to moderate dyspnea. Continued exposure may lead to progressive fi brosis and severe respiratory embarrassment. Anthracosis (choice A) by itself does not cause restrictive lung disease, whereas asbestosis (choice B) is characterized by interstitial fi brosis. The nodules of sarcoidosis (choice C) and Wegener granulomatosis (choice E) are not fi brotic. Diagnosis: Silicosis

A 60-year-old mason complains of shortness of breath, which has become progressively worse during the past year. A chest X-ray shows small nodular shadows in both lungs. Pulmonary function studies reveal a pattern consistent with restrictive lung disease. The patient subsequently develops congestive heart failure and expires. Autopsy discloses numerous small, fi brotic nodules in both lungs. Histologic examination of these nodules is shown in the image. Which of the following is the most likely diagnosis (A) Anthracosis (B) Asbestosis (C) Sarcoidosis (D) Silicosis (E) Wegener granulomatosis

the answer is D. Chronic bronchitis is an example of COPD. The pathologic hallmark of chronic bronchitis is marked hyperplasia of bronchial submucosal glands and bronchial smooth muscle hypertrophy, which can be quantified by the Reid index, a ratio of glandu-lar layer thickness to bronchial wall thickness

A 60-year-old woman with a heavy smok-ing history presents with chronic productive cough that has been present for three con-secutive months over the past two consecu-tive years. On physical examination, her skin has a bluish tinge, and she is overweight. The patient is diagnosed with chronic bronchitis. Which of the following is the most likely his-tologic finding in this patient's lungs? (a)Bronchial smooth muscle hypertrophy with proliferation of eosinophils (B)Diffuse alveolar damage with leakage of protein-rich fluid into alveolar spaces (c)Dilation of air spaces with destruction of alveolar walls (D)Hyperplasia of bronchial mucus-secreting submucosal glands (e)Permanent bronchial dilation caused by chronic infection, with bronchi filled with mucus and neutrophils

The answer is E: Diff use alveolar damage (DAD). DAD refers to a nonspecifi c pattern of reaction to injury of alveolar epi-thelial and endothelial cells from a variety of acute insults. The clinical counterpart of severe DAD is acute respiratory distress syndrome. In this disorder, a patient with apparently normal lungs sustains pulmonary damage and then develops rapid progressive respiratory failure. DAD is a fi nal common pathway of pathologic changes caused by a variety of insults, including respiratory infections, sepsis, shock, aspiration of gastric con-tents, inhalation of toxic gases, near-drowning, radiation pneu-monitis, and a large assortment of drugs and other chemicals. Desquamative interstitial pneumonia (choice D) is a chronic, fi brosing, interstitial pneumonitis of unknown etiology. Diagnosis: Diffuse alveolar damage

A 62-year-old woman is rushed to the emergency room following an automobile accident. She has suffered internal injuries and massive bleeding and appears to be in a state of profound shock. Her temperature is 37°C (98.6°F), respira-tions are 42 per minute, and blood pressure is 80/40 mm Hg. Physical examination shows cyanosis and the use of acces-sory respiratory muscles. A CT scan of the chest is normal on arrival. Her condition is complicated by fever, leukocytosis, and a positive blood culture for staphylococci (sepsis). Two days later, the patient develops rapidly progressive respiratory distress, and a pattern of "interstitial pneumonia" can be seen on a chest X-ray. Which of the following is the most likely diagnosis? (A) Acute bronchiolitis (B) Alveolar proteinosis (C) Atelectasis (D) Desquamative interstitial pneumonitis (E) Diffuse alveolar damage

The answer is c. An individual who has trouble breathing, and while sitting leans forward slightly and breathes quicklythrough pursed-lips, and is also found to have an increased anteroposterior diameter to their chest ("barrel chest") most likely has emphysema. This abnormality, whichresults from abnormal dilation of the air spaces distal to the terminal bronchioles, may be classified based on the anatomic location of the abnormal dilation within therespiratory lobule. The normal respiratory lobule is composed of three to five terminal bronchioles and their acini, which in turn are composed of a respiratory bronchiole,alveolar ducts, alveolar sacs, and alveoli. Emphysema may affect the proximal acinus (centrilobular emphysema), the distal acinus (paraseptal emphysema), or the entireacinus (panlobular emphysema). It is postulated that emphysema results from an imbalance between elastase, which is produced by neutrophils and macrophages and destroysthe walls of airways, and antielastase, which inactivates elastase. Cigarette smoking, which is associated with the production of centrilobular emphysema, increases elastaseactivity and decreases α1 antitrypsin

A 63-year-old man who is a long-term smoker presents with increasing shortness of breath and dyspnea. He has smoked more than two packs of cigarettes per day formore than 40 years. He denies having a productive cough or any recent infections. Physical examination reveals a thin elderly appearing man in moderate respiratorydistress. While sitting he leans slightly forward and breathes quickly through pursed lips. He is afebrile and his blood pressure is within normal limits. Examination of his chestreveals an increased anteroposterior diameter and his lungs are hyper-resonant to percussion. His respiratory rate is increased, but no clubbing or cyanosis is present. Chest x-ray reveals his heart to be of normal size, but there is hyperinflation of his lungs. Laboratory examination reveals that while breathing room air, his arterial PO2 is decreasedbut his arterial PCO2 is normal. Which of the following statements is an accurate association concerning the pathogenesis of this man's pulmonary disease? a. Destruction of entire acinus caused panlobular emphysema b. Destruction of the cilia on the respiratory epithelial cells resulted in bronchiectasis c. Destruction of the proximal acinus caused centrilobular emphysema d. Hyperplasia of the respiratory smooth-muscle cells resulted in intrinsic asthma e. Hyperplasia on the respiratory mucus glands caused chronic bronchitis

The answer is B: Disorganized normal tissue. Localized, dis-ordered differentiation during embryonic development results in a hamartoma, a disorganized caricature of normal tissue components. Such tumors, which are not strictly neoplasms, contain varying combinations of cartilage, ducts or bronchi, connective tissue, blood vessels, and lymphoid tissue. Ectopic islands of normal tissue (choice C), called choristoma, may also be mistaken for true neoplasms. These small lesions are represented by pancreatic tissue in the wall of the stomach or intestine, adrenal rests under the renal capsule, and nodules of splenic tissue in the peritoneal cavity. Diagnosis: Hamartom

A 63-year-old woman with chronic bronchitis presents with shortness of breath. A chest X-ray reveals a 2-cm "coin lesion" in the upper lobe of the left lung. A CT-guided lung biopsy is obtained. Which of the following describes the histologic features of this lesion if the diagnosis is hamartoma? (A) Benign neoplasm of epithelial origin (B) Disorganized normal tissue (C) Ectopic islands of normal tissue (D) Granulation tissue (E) Granulomatous infl ammation

The answer is D: Malignant mesothelioma. Mesothelioma is a malignant neoplasm of mesothelial cells that is most common in the pleura, but also occurs in the peritoneum, pericardium, and the tunica vaginalis of the testis. The tumor is strongly linked to occupational inhalation of asbes-tos. Patients are often fi rst seen with a pleural effusion or a pleural mass, chest pain, and nonspecifi c symptoms, such as weight loss and malaise. Pleural mesotheliomas tend to spread locally and extensively within the chest cavity, but do not typically invade the pulmonary parenchyma. Wide-spread metastases can occur. Mesothelioma is typically composed of both epithelial and sarcomatous elements (i.e., biphasic pattern). The other choices do not ordinarily encase the lung. Diagnosis: Malignant mesotheliom

A 64-year-old man who has worked in a manufacturing plant all his life complains of an 8-month history of chest discom-fort, malaise, fever, night sweats, and weight loss. A chest X-ray reveals a pleural effusion and pleural mass encasing the lung. The patient subsequently dies of cardiorespiratory failure. Histologic examination of the pleural mass at autopsy shows a biphasic pattern of epithelial and sarcomatous ele-ments. What is the most likely diagnosis? (A) Carcinoid tumor (B) Large cell carcinoma (C) Malignant melanoma (D) Malignant mesothelioma (E) Metastatic carcinoma

The answer is A: Anthracosilicosis. Coal dust is composed of amorphous carbon and other constituents of the earth's sur-face, including variable amounts of silica. Amorphous carbon by itself is not fi brogenic owing to its inability to kill alveo-lar macrophages. It is simply a nuisance dust that causes an innocuous anthracosis. By contrast, silica is highly fi brogenic, and the inhalation of rock particles may therefore lead to the lesions of anthracosilicosis. Coal workers' pneumoconiosis is also known as "black lung disease" due to massive deposits of carbon particles. The characteristic pulmonary lesions of com-plicated coal workers' pneumoconiosis include palpable coal-dust nodules scattered throughout the lung as 1- to 4-mm black foci. Nodules consist of dust-laden macrophages associ-ated with a fi brotic stroma. Coal miners are not predisposed to the other choices. Diagnosis: Coal workers' pneumoconiosis, anthracosilicosis

A 65-year-old coal miner is admitted for evaluation of chronic lung disease. The patient admits to smoking one pack of cigarettes a day for 40 years. On physical examination, he is noticed to have a barrel chest and use accessory muscles for inspiration. His face is puffy and red. He has 2+ pitting edema of the lower extremities. A chest X-ray is compatible with dif-fuse fi brosis, with some nodularity in central areas. Which of the following is the most likely diagnosis? (A) Anthracosilicosis (B) Asbestosis (C) Diffuse alveolar damage (D) Psittacosis (E) Sarcoidosis

The answer is B: Paraneoplastic syndrome. Cancers may pro-duce remote effects, collectively termed paraneoplastic syn-dromes. For example, the secretion of corticotropin (ACTH) by a tumor leads to clinical features of Cushing syndrome, including hyperglycemia and hypertension. Corticotropin production is most commonly seen with cancers of the lung, particularly small cell carcinoma. Adrenal and pituitary metas-tases (choices A and D) would lead to loss of adrenal function (Addison disease). Although pituitary adenoma (choice C) is a possible cause of Cushing syndrome, this choice would be unlikely in a patient with lung cancer. Diagnosis: Small cell carcinoma of lung, paraneoplastic syndrome

A 65-year-old man complains of muscle weakness and a dry cough for 4 months. He has smoked two packs of cigarettes daily for 45 years. A chest X-ray shows a 4-cm central, left lung mass. Laboratory studies reveal hyperglycemia and hypertension. A transbronchial biopsy is diagnosed as small cell carcinoma. Metastases to the liver are detected by CT scan. Which of the following might account for the development of hyperglycemia and hypertension in this patient? (A) Adrenal metastases (B) Paraneoplastic syndrome (C) Pituitary adenoma (D) Pituitary metastases (E) Thrombosis of the renal artery

The answer is c. The segmented or beaded, often dumbbell-shaped bodies are ferruginous bodiesthat are probably asbestos fibers coated with iron and protein. The term ferruginous body is applied to other inhaled fibers that become iron coated; however, in a patient withinterstitial lung fibrosis or pleural plaques, ferruginous bodies are probably asbestos bodies. The type of asbestos mainly used in America is chrysotile, mined in Canada, and itis much less likely to cause mesothelioma or lung cancer than is crocidolite (blue asbestos), which has limited use and is mined in South Africa. Cigarette smoking potentiates the relatively mild carcinogenic effect of asbestos. In contrast, laminated spherical (Schaumann) bodies are found in granulomas of sarcoid and chronic berylliosis, whileCandida species histologically may show elongated chains of yeast without hyphae (pseudohyphae), and silica particles are very small and are birefringent

A 65-year-old man who just retired after having worked for many years in a shipyard presents with increasing shortness of breath. Pertinent medical history is that hehas been a long-time smoker. A CT scan of his chest reveals thick, pleural plaques on the surface of his lungs. The associated picture is from a bronchial washing specimenfrom this patient. The dumbbell-shaped structures in this picture were found to stain blue with a Prussian blue stain. What are these structures? a. Candida species b. Cholesterol crystals c. Ferruginous bodies d. Schaumann bodies e. Silica particles

the answer is D. Small cell carcinoma of the lung is the most aggressive type of broncho-genic carcinoma. The location of this cancer is usually central. This is an undifferentiated tumor with small round blue cells and is least likely to be cured by surgery because it is usually already metastatic at diagnosis. Associated paraneoplastic syndromes include secretion of adrenocorticotropic hormone and antidiuretic hormone.

A 65-year-old woman with a significant smoking history presents with cough and shortness of breath. Computed tomography of the chest reveals a central mass near the left mainstem bronchus. Biopsy of the mass is performed. Histologic examination reveals small round blue cells, and a diagnosis of small cell carcinoma is made. Which of the following is a frequent characteristic of this form of lung cancer? (a)Generally amenable to surgical cure at time of diagnosis (B)More common in women, and a less clear relation to smoking than other forms of lung cancer (c)Secretes a parathyroid-like hormone (D)Secretes either corticotrophin or antidi-uretic hormone (e)Usually in a peripheral rather than in a central location

The answer is d. Lung cancers are classified according to their histologic appearance. First, they are divided into two groups based on the size of the tumor cells, namely, small-cell carcinomas and non-small-cell carcinomas. Small-cell carcinomas, also called "oat cell" carcinomas, contain scantamounts of cytoplasm, and their nuclei are small and round and rarely have nucleoli. These malignancies, which are of neuroendocrine origin and display neurosecretorygranules on electron microscopy, may cause a variety of paraneoplastic syndromes, such as from the synthesis and secretion of hormones such as ACTH and serotonin. Othereffects not well understood on the neuromuscular system include central encephalopathy and Eaton-Lambert syndrome, a myasthenic syndrome resulting from impairedrelease of acetylcholine and usually associated with pulmonary oat cell carcinoma. Oat cell carcinomas form 20% to 25% of primary lung tumors, occur most frequently inmen of middle age or older, have a strong association with cigarette smoking, and carry a poor prognosis, as they metastasize early (eg, bone marrow metastases). The non-small-cell carcinomas are classified as

A 67-year-old man who is a long-term smoker presents with weight loss, a persistent cough, fever, chest pain, and hemoptysis. Physical examination reveals a cachecticman with clubbing of his fingers and dullness to percussion over his right lower lobe. A chest x-ray reveals a 3.5-cm hilar mass on the right and postobstructive pneumonia ofthe right lower lobe. Sputum cytology is suspicious for malignant cells. Histologic examination of a transbronchial biopsy specimen reveals infiltrating groups of cells withscant cytoplasm. No glandular structures or keratin production are seen. The nuclei of these cells are about twice the size of normal lymphocytes and do not appear to havenucleoli. Which of the following is the most likely diagnosis? a. Adenocarcinoma b. Hamartoma c. Large-cell undifferentiated carcinoma d. Small-cell undifferentiated carcinoma e. Squamous cell carcinoma

The answer is B. Pleural and peritoneal mesotheliomas are associated with exposure to asbestos, and the apparent tumorigenic effect of asbestosis is markedly enhanced by cigarette smoking. Aflatoxin B1 is associated with hepatocellular carcinoma. Clear cell adenocarcinoma of the vagina has been a hazard to daughters exposed during intrauterine life to DES administered to their mothers to prevent spontaneous abortion. Ionizing radiation is associated with many cancers, including leukemias, breast cancer, and thyroid malignancies. β-naphthylamine and other aniline dyes are associated with transitional cell carcinoma of the bladder.

A 67-year-old man, a heavy smoker, is seen because of dyspnea and cough. AchestX-ray reveals abnormal densities, and a computed tomography (CT) scan is suggestive of a neoplasm involving the pleura. A biopsy confirms the diagnosis of mesothelioma. Other than cigarette smoke, this finding suggests exposure to which toxin? (A)Aflatoxin B1 (B)Asbestos (C)Diethylstilbestrol (DES) (D)Ionizing radiation (E)β-Naphthylamine

The answer is E: Squamous cell carcinoma. Squamous cell carcinoma accounts for 30% of all invasive lung cancers in the United States. Well-differentiated squamous cell carcinoma displays keratin "pearls," which appear as a small round nest of brightly eosinophilic aggregates of keratin surrounded by concentric ("onion skin") layers of squamous cells. Gland for-mation is exhibited in adenocarcinoma (choices A and C). Diagnosis: Squamous cell carcinoma of lung

A 68-year-old man complains of shortness of breath, hoarse-ness, productive cough, and bloody sputum of 2 weeks in dura-tion. He admits to smoking two packs a day for 45 years and drinks occasionally. Recently, he has experienced a signifi cant loss of appetite and weight loss. Physical examination shows pallor, cachexia, clubbing of the fi ngers, and barrel-shaped chest. A chest X-ray reveals a mass at the right lung apex. His-tologic examination of a transbronchial biopsy is shown in the image. What is the appropriate histologic diagnosis? (A) Adenocarcinoma (B) Mesothelioma (C) Metastatic adenocarcinoma (D) Small cell carcinoma (E) Squamous cell carcinoma

The answer is B: Asbestos. The characteristic tumor asso-ciated with asbestos exposure is mesothelioma of the pleu-ral and peritoneal cavities. This cancer has been reported to occur in 2% to 3% of heavily exposed workers. The pipe fi t-ters in shipyards were the most exposed workers. Many of these workers developed mesotheliomas 20 to 40 years after exposure. It is reasonable to surmise that mesotheliomas of both the pleura and the peritoneum refl ect the close contof these membranes with asbestos fi bers transported to them by lymphatic channels. Like the polycyclic aromatic hydro-carbons, afl atoxin B1 (choice A) can bind covalently to DNA and is among the most potent liver carcinogens recognized. Beryllium (choice C) and silica (choice E) cause lung disease, but they are not carcinogenic. Diagnosis: Mesothelioma

A 68-year-old man who has worked in a shipyard and manu-facturing plant all his adult life complains of a 4-month history of chest discomfort, malaise, fever, night sweats, and weight loss. A chest X-ray reveals a large pleural effusion. The patient dies 5 months later of cardiorespiratory failure. The lung at autopsy is shown in the image. This malignant neoplasm is associated with environmental exposure to which of the fol-lowing carcinogens? (A) Afl atoxin B1 (B) Asbestos (C) Beryllium (D) Ionizing radiation (E) Silica

the answer is B. Tobacco contributes to the development of many cancers, including those of the bladder, lung, throat, and esophagus. Asbestos exposure carries a risk of lung cancer, as well as mesothelioma, a cancer of the pleura. Tobacco and asbestos func-tion as cocarcinogens in the pathogenesis of lung cancer, with an approximately 50-fold greater risk of developing bronchogenic cancer of the lung than in those without such exposure

A 78-year-old Navy veteran with a 600-pack-per-year history of cigarette smok-ing presents with cancer. During his mili-tary career, he was involved in fireproofing naval combat ships with asbestos insulation. Given his environmental exposure to both tobacco and asbestos, to which cancer do both of these carcinogens contribute? (a)Bladder cancer (B)Bronchogenic cancer (C)Cancer of the throat (d)Esophageal cancer (e)Mesothelioma

The answer is e. Tuberculosis is more common in individuals with silicosis, a type of pneumoconiosis seen in sandblastersand mine workers, which is perhaps the most common chronic occupational disease in the world. The pneumoconioses are pulmonary diseases that are caused bynonneoplastic lung reactions to several types of environmental dusts. The characteristic pathology of silicosis is pulmonary fibrosis. Early in the disease there are multiple,very small nodules in the upper zones of the lung, which produces a fine nodularity on x-ray. These areas histologically show fibrosis and birefringent particles. The fibroticlesions may also be found in the hilar lymph nodes, which can become calcified and have an "eggshell" pattern on x-ray examination. Silica depresses cell-mediated immunity,and crystalline silica may inhibit pulmonary macrophages, which may be why individuals with silicosis are at a high risk for developing tuberculosis. There is no clearrelationship between silicosis and the development of cancer. This is in contrast to asbestosis, which is associated with the development of lung cancer and mesothelioma.

A routine chest x-ray performed on an asymptomatic 31-year-old man who works at sandblasting reveals a fine nodularity in the upper zones of the lungs and "eggshell"calcification of the hilar lymph nodes. The patient's serum calcium level is 9.8 mg/dL, while his total protein is 7.2 g/dL. He denies any history of drug use or cigarettesmoking. A biopsy from his lung reveals birefringent particles within macrophages. This individual has an increased risk for developing which one of the listed disorders? a. Anthracosis b. Berylliosis c. Myxomatosis d. Sarcoidosis e. Tuberculosis

The answer is B: Asthma. Asthma is a chronic lung disease caused by increased responsiveness of the airways to a variety of stimuli. Patients typically have paroxysms of wheezing, dyspnea, and cough. Acute episodes of asthma may alternate with asymp-tomatic periods or they may be superimposed on a background of chronic airway obstruction. The consensus hypothesis attri-butes bronchial hyperresponsiveness in asthma to an infl amma-tory reaction to diverse stimuli, either extrinsic (e.g., pollen) or intrinsic (e.g., exercise). Extrinsic asthma is typically a childhood disease, whereas intrinsic asthma usually begins in adults. The other choices do not lead to wheezing and eosinophilia. Diagnosis: Asthma

An 8-year-old girl is brought into the physician's offi ce in mild respiratory distress. She has a history of allergies to cats and wool, and her parents state that she has recurrent epi-sodes of upper respiratory tract infections. Physical examina-tion shows expiratory wheezes, use of accessory respiratory muscles, and a hyperresonant chest to percussion. Analysis of arterial blood gases discloses respiratory alkalosis, and the peripheral eosinophil count is increased. What is the appro-priate diagnosis? (A) Acute bronchiolitis (B) Asthma (C) Cystic fi brosis (D) Kartagener syndrome (E) Usual interstitial pneumonia

The answer is e. Pulmonary hamartomas, although infrequent, are still the most common of all benign lung tumors.Hamartomas consist of various tissues normally found in the organ where they develop, but in abnormal amounts and arrangements. In the lung they consist of lobules ofconnective tissue often containing mature cartilage, fat, or fibrous tissue and separated by clefts lined by entrapped respiratory epithelium. The peak incidence is at age 60,and the tumor is usually found as a well-circumscribed, peripheral "coin" lesion on routine chest x-ray. Unless the radiographic findings are pathognomonic of hamartomawith "popcorn ball" calcifications, the lesion should be excised or at least carefully followed.

During a routine physical examination, a 43-year-old man is found to have a 2.5-cm "coin" in the peripheral portion of his right upper lobe (RUL). Several sputumsamples sent for cytology are unremarkable, and a bronchoscopic examination is also unremarkable. Surgery is performed and the mass is resected. Histologic examinationreveals lobules of connective tissue that contain mature hyaline cartilage. These lobules are separated by clefts that are lined by respiratory epithelium. Which of thefollowing is the most likely diagnosis? a. Adenocarcinoma b. Bronchioloalveolar carcinoma c. Carcinoid d. Fibroma e. Hamartoma

The answer is e. C-ANCAs, or cytoplasmic-staining antineutrophil cytoplasmicantibodies, are antibodies that are directed against neutral leukocyte protease (proteinase 3). This pattern is typically associated with Wegener granulomatosis, which ischaracterized clinically by the triad of acute necrotizing granulomas of the upper and lower respiratory tract, necrotizing or granulomatous vasculitis affecting small- tomedium-sized vessels, and renal disease.C-ANCAs are also useful in evaluating the treatment response of patients with Wegener granulomatosis. C-ANCAs are present in the serum in the majority of patients withactive disease. Most patients in remission have a negative test, while a rising titer of C-ANCA during treatment suggests a relapse of the disease. The otherimmunofluorescence pattern of ANCAs is directed against myeloperoxidase of neutrophils and is found in a perinuclear location (P-ANCAs). This pattern is seen in patientswith microscopic PAN (microscopic polyangiitis), idiopathic crescentic glomerulonephritis without systemic disease, or Churg-Strauss syndrome. Microscopic PANcommonly involves glomeruli

In a patient with vasculitis, the finding of serum antineutrophil cytoplasmic autoantibodies (ANCAs) that react by immunofluorescence staining in a cytoplasmicstaining pattern (C-ANCAs) is most suggestive of which one of the following diseases? a. Classic polyarteritis nodosa b. Goodpasture disease c. Henoch-Schönlein purpura d. Microscopic polyangiitis e. Wegener granulomatosis


संबंधित स्टडी सेट्स

Causes of exchange rate changes - Exchange rates change whenever there is a change (increase or decrease) in the demand or supply of a currency

View Set

POSHER PRAMS and sentence types for Paper 1, Question 2

View Set

Tagalog - 625 Most Important Words - By Category

View Set

4.3.1: Understanding the Natural Rate of Unemployment

View Set